Proving linear independence and that $dim V$ is greater than or equals 3.












1














Let $V= operatorname{span}{v_1,v_2,v_3,v_4}$ be a vector space such that $v_i$ are unit vectors for all $i$ and $v_i.v_j<0$ if $i$ does not equals to $j$.



(i) Show that no two vectors among {$v_1,v_2,v_3,v_4$} are linearly dependent.



(ii) Prove that $dim V$ is greater than or equals to 3.



My approach:
(i) If all the vectors are linearly dependent, this would mean that $v_i=av_j$.
However, since |$v_i$| = 1, it does not = a|$v_j$| unless $a = 1$. Thus, it can be proven that no two vectors in the set are linearly dependent.



Please correct me if my approach is incorrect, and may I ask how do I approach the second part of the question?










share|cite|improve this question




















  • 1




    In 1) you only get $|a|=1$ not $a=1$. So $a =pm 1$, If $a=-1$ you have to use the fact that $v_3.v_1<0$ and $v_3.v_2<0$ to get a contradiction.
    – Kavi Rama Murthy
    Nov 27 '18 at 6:33


















1














Let $V= operatorname{span}{v_1,v_2,v_3,v_4}$ be a vector space such that $v_i$ are unit vectors for all $i$ and $v_i.v_j<0$ if $i$ does not equals to $j$.



(i) Show that no two vectors among {$v_1,v_2,v_3,v_4$} are linearly dependent.



(ii) Prove that $dim V$ is greater than or equals to 3.



My approach:
(i) If all the vectors are linearly dependent, this would mean that $v_i=av_j$.
However, since |$v_i$| = 1, it does not = a|$v_j$| unless $a = 1$. Thus, it can be proven that no two vectors in the set are linearly dependent.



Please correct me if my approach is incorrect, and may I ask how do I approach the second part of the question?










share|cite|improve this question




















  • 1




    In 1) you only get $|a|=1$ not $a=1$. So $a =pm 1$, If $a=-1$ you have to use the fact that $v_3.v_1<0$ and $v_3.v_2<0$ to get a contradiction.
    – Kavi Rama Murthy
    Nov 27 '18 at 6:33
















1












1








1







Let $V= operatorname{span}{v_1,v_2,v_3,v_4}$ be a vector space such that $v_i$ are unit vectors for all $i$ and $v_i.v_j<0$ if $i$ does not equals to $j$.



(i) Show that no two vectors among {$v_1,v_2,v_3,v_4$} are linearly dependent.



(ii) Prove that $dim V$ is greater than or equals to 3.



My approach:
(i) If all the vectors are linearly dependent, this would mean that $v_i=av_j$.
However, since |$v_i$| = 1, it does not = a|$v_j$| unless $a = 1$. Thus, it can be proven that no two vectors in the set are linearly dependent.



Please correct me if my approach is incorrect, and may I ask how do I approach the second part of the question?










share|cite|improve this question















Let $V= operatorname{span}{v_1,v_2,v_3,v_4}$ be a vector space such that $v_i$ are unit vectors for all $i$ and $v_i.v_j<0$ if $i$ does not equals to $j$.



(i) Show that no two vectors among {$v_1,v_2,v_3,v_4$} are linearly dependent.



(ii) Prove that $dim V$ is greater than or equals to 3.



My approach:
(i) If all the vectors are linearly dependent, this would mean that $v_i=av_j$.
However, since |$v_i$| = 1, it does not = a|$v_j$| unless $a = 1$. Thus, it can be proven that no two vectors in the set are linearly dependent.



Please correct me if my approach is incorrect, and may I ask how do I approach the second part of the question?







linear-algebra matrices eigenvalues-eigenvectors independence






share|cite|improve this question















share|cite|improve this question













share|cite|improve this question




share|cite|improve this question








edited Nov 27 '18 at 10:17









Bernard

118k639112




118k639112










asked Nov 27 '18 at 6:20









Cheryl

755




755








  • 1




    In 1) you only get $|a|=1$ not $a=1$. So $a =pm 1$, If $a=-1$ you have to use the fact that $v_3.v_1<0$ and $v_3.v_2<0$ to get a contradiction.
    – Kavi Rama Murthy
    Nov 27 '18 at 6:33
















  • 1




    In 1) you only get $|a|=1$ not $a=1$. So $a =pm 1$, If $a=-1$ you have to use the fact that $v_3.v_1<0$ and $v_3.v_2<0$ to get a contradiction.
    – Kavi Rama Murthy
    Nov 27 '18 at 6:33










1




1




In 1) you only get $|a|=1$ not $a=1$. So $a =pm 1$, If $a=-1$ you have to use the fact that $v_3.v_1<0$ and $v_3.v_2<0$ to get a contradiction.
– Kavi Rama Murthy
Nov 27 '18 at 6:33






In 1) you only get $|a|=1$ not $a=1$. So $a =pm 1$, If $a=-1$ you have to use the fact that $v_3.v_1<0$ and $v_3.v_2<0$ to get a contradiction.
– Kavi Rama Murthy
Nov 27 '18 at 6:33












1 Answer
1






active

oldest

votes


















0














(i) Assume that exists $i,jin{1,2,3,4}$ such that $v_{i}=av_{j}$. Considering the norm, we have $1=||v_{i}||=||av_{j}||=|a|,||v_{j}||=|a| Rightarrow a=-1,$, since $a=v_{i}cdot v_{j}<0$. However, we have $v_{i}cdot v_{k}=a v_{j}cdot v_{k}=- v_{j}cdot v_{k} > 0$ for $kneq i,j$. This is absurd.



(ii) Thanks to (i) $,operatorname{dim} V >1$, so it is sufficient to prove that $operatorname{dim} V neq 2$.

We assume for absurd that $operatorname{dim} V = 2$.
Let $w= v_{2} - frac{(v_{2}cdot v_{1})}{||v_{1}||}, v_{1} in V$ such that $wcdot v_{1}=0$. Then $operatorname{span}{v_{1},v_{2},v_{3},v_{4}}=operatorname{span}{v_{1},w}$, and we can write
$$v_{3}=a_{1}v_{1}+a_{2}w; ,; v_{4}=b_{1}v_{1}+b_{2}w ;;text{with}; a_{i},b_{i}<0$$
but $0>v_{3}cdot v_{4}=a_{1}b_{1}+a_{2}b_{2}>0$.

Follow that $operatorname{dim}Vgeq 3$.






share|cite|improve this answer























    Your Answer





    StackExchange.ifUsing("editor", function () {
    return StackExchange.using("mathjaxEditing", function () {
    StackExchange.MarkdownEditor.creationCallbacks.add(function (editor, postfix) {
    StackExchange.mathjaxEditing.prepareWmdForMathJax(editor, postfix, [["$", "$"], ["\\(","\\)"]]);
    });
    });
    }, "mathjax-editing");

    StackExchange.ready(function() {
    var channelOptions = {
    tags: "".split(" "),
    id: "69"
    };
    initTagRenderer("".split(" "), "".split(" "), channelOptions);

    StackExchange.using("externalEditor", function() {
    // Have to fire editor after snippets, if snippets enabled
    if (StackExchange.settings.snippets.snippetsEnabled) {
    StackExchange.using("snippets", function() {
    createEditor();
    });
    }
    else {
    createEditor();
    }
    });

    function createEditor() {
    StackExchange.prepareEditor({
    heartbeatType: 'answer',
    autoActivateHeartbeat: false,
    convertImagesToLinks: true,
    noModals: true,
    showLowRepImageUploadWarning: true,
    reputationToPostImages: 10,
    bindNavPrevention: true,
    postfix: "",
    imageUploader: {
    brandingHtml: "Powered by u003ca class="icon-imgur-white" href="https://imgur.com/"u003eu003c/au003e",
    contentPolicyHtml: "User contributions licensed under u003ca href="https://creativecommons.org/licenses/by-sa/3.0/"u003ecc by-sa 3.0 with attribution requiredu003c/au003e u003ca href="https://stackoverflow.com/legal/content-policy"u003e(content policy)u003c/au003e",
    allowUrls: true
    },
    noCode: true, onDemand: true,
    discardSelector: ".discard-answer"
    ,immediatelyShowMarkdownHelp:true
    });


    }
    });














    draft saved

    draft discarded


















    StackExchange.ready(
    function () {
    StackExchange.openid.initPostLogin('.new-post-login', 'https%3a%2f%2fmath.stackexchange.com%2fquestions%2f3015408%2fproving-linear-independence-and-that-dim-v-is-greater-than-or-equals-3%23new-answer', 'question_page');
    }
    );

    Post as a guest















    Required, but never shown

























    1 Answer
    1






    active

    oldest

    votes








    1 Answer
    1






    active

    oldest

    votes









    active

    oldest

    votes






    active

    oldest

    votes









    0














    (i) Assume that exists $i,jin{1,2,3,4}$ such that $v_{i}=av_{j}$. Considering the norm, we have $1=||v_{i}||=||av_{j}||=|a|,||v_{j}||=|a| Rightarrow a=-1,$, since $a=v_{i}cdot v_{j}<0$. However, we have $v_{i}cdot v_{k}=a v_{j}cdot v_{k}=- v_{j}cdot v_{k} > 0$ for $kneq i,j$. This is absurd.



    (ii) Thanks to (i) $,operatorname{dim} V >1$, so it is sufficient to prove that $operatorname{dim} V neq 2$.

    We assume for absurd that $operatorname{dim} V = 2$.
    Let $w= v_{2} - frac{(v_{2}cdot v_{1})}{||v_{1}||}, v_{1} in V$ such that $wcdot v_{1}=0$. Then $operatorname{span}{v_{1},v_{2},v_{3},v_{4}}=operatorname{span}{v_{1},w}$, and we can write
    $$v_{3}=a_{1}v_{1}+a_{2}w; ,; v_{4}=b_{1}v_{1}+b_{2}w ;;text{with}; a_{i},b_{i}<0$$
    but $0>v_{3}cdot v_{4}=a_{1}b_{1}+a_{2}b_{2}>0$.

    Follow that $operatorname{dim}Vgeq 3$.






    share|cite|improve this answer




























      0














      (i) Assume that exists $i,jin{1,2,3,4}$ such that $v_{i}=av_{j}$. Considering the norm, we have $1=||v_{i}||=||av_{j}||=|a|,||v_{j}||=|a| Rightarrow a=-1,$, since $a=v_{i}cdot v_{j}<0$. However, we have $v_{i}cdot v_{k}=a v_{j}cdot v_{k}=- v_{j}cdot v_{k} > 0$ for $kneq i,j$. This is absurd.



      (ii) Thanks to (i) $,operatorname{dim} V >1$, so it is sufficient to prove that $operatorname{dim} V neq 2$.

      We assume for absurd that $operatorname{dim} V = 2$.
      Let $w= v_{2} - frac{(v_{2}cdot v_{1})}{||v_{1}||}, v_{1} in V$ such that $wcdot v_{1}=0$. Then $operatorname{span}{v_{1},v_{2},v_{3},v_{4}}=operatorname{span}{v_{1},w}$, and we can write
      $$v_{3}=a_{1}v_{1}+a_{2}w; ,; v_{4}=b_{1}v_{1}+b_{2}w ;;text{with}; a_{i},b_{i}<0$$
      but $0>v_{3}cdot v_{4}=a_{1}b_{1}+a_{2}b_{2}>0$.

      Follow that $operatorname{dim}Vgeq 3$.






      share|cite|improve this answer


























        0












        0








        0






        (i) Assume that exists $i,jin{1,2,3,4}$ such that $v_{i}=av_{j}$. Considering the norm, we have $1=||v_{i}||=||av_{j}||=|a|,||v_{j}||=|a| Rightarrow a=-1,$, since $a=v_{i}cdot v_{j}<0$. However, we have $v_{i}cdot v_{k}=a v_{j}cdot v_{k}=- v_{j}cdot v_{k} > 0$ for $kneq i,j$. This is absurd.



        (ii) Thanks to (i) $,operatorname{dim} V >1$, so it is sufficient to prove that $operatorname{dim} V neq 2$.

        We assume for absurd that $operatorname{dim} V = 2$.
        Let $w= v_{2} - frac{(v_{2}cdot v_{1})}{||v_{1}||}, v_{1} in V$ such that $wcdot v_{1}=0$. Then $operatorname{span}{v_{1},v_{2},v_{3},v_{4}}=operatorname{span}{v_{1},w}$, and we can write
        $$v_{3}=a_{1}v_{1}+a_{2}w; ,; v_{4}=b_{1}v_{1}+b_{2}w ;;text{with}; a_{i},b_{i}<0$$
        but $0>v_{3}cdot v_{4}=a_{1}b_{1}+a_{2}b_{2}>0$.

        Follow that $operatorname{dim}Vgeq 3$.






        share|cite|improve this answer














        (i) Assume that exists $i,jin{1,2,3,4}$ such that $v_{i}=av_{j}$. Considering the norm, we have $1=||v_{i}||=||av_{j}||=|a|,||v_{j}||=|a| Rightarrow a=-1,$, since $a=v_{i}cdot v_{j}<0$. However, we have $v_{i}cdot v_{k}=a v_{j}cdot v_{k}=- v_{j}cdot v_{k} > 0$ for $kneq i,j$. This is absurd.



        (ii) Thanks to (i) $,operatorname{dim} V >1$, so it is sufficient to prove that $operatorname{dim} V neq 2$.

        We assume for absurd that $operatorname{dim} V = 2$.
        Let $w= v_{2} - frac{(v_{2}cdot v_{1})}{||v_{1}||}, v_{1} in V$ such that $wcdot v_{1}=0$. Then $operatorname{span}{v_{1},v_{2},v_{3},v_{4}}=operatorname{span}{v_{1},w}$, and we can write
        $$v_{3}=a_{1}v_{1}+a_{2}w; ,; v_{4}=b_{1}v_{1}+b_{2}w ;;text{with}; a_{i},b_{i}<0$$
        but $0>v_{3}cdot v_{4}=a_{1}b_{1}+a_{2}b_{2}>0$.

        Follow that $operatorname{dim}Vgeq 3$.







        share|cite|improve this answer














        share|cite|improve this answer



        share|cite|improve this answer








        edited Nov 27 '18 at 10:23

























        answered Nov 27 '18 at 10:08









        Andrea Cavaliere

        1056




        1056






























            draft saved

            draft discarded




















































            Thanks for contributing an answer to Mathematics Stack Exchange!


            • Please be sure to answer the question. Provide details and share your research!

            But avoid



            • Asking for help, clarification, or responding to other answers.

            • Making statements based on opinion; back them up with references or personal experience.


            Use MathJax to format equations. MathJax reference.


            To learn more, see our tips on writing great answers.





            Some of your past answers have not been well-received, and you're in danger of being blocked from answering.


            Please pay close attention to the following guidance:


            • Please be sure to answer the question. Provide details and share your research!

            But avoid



            • Asking for help, clarification, or responding to other answers.

            • Making statements based on opinion; back them up with references or personal experience.


            To learn more, see our tips on writing great answers.




            draft saved


            draft discarded














            StackExchange.ready(
            function () {
            StackExchange.openid.initPostLogin('.new-post-login', 'https%3a%2f%2fmath.stackexchange.com%2fquestions%2f3015408%2fproving-linear-independence-and-that-dim-v-is-greater-than-or-equals-3%23new-answer', 'question_page');
            }
            );

            Post as a guest















            Required, but never shown





















































            Required, but never shown














            Required, but never shown












            Required, but never shown







            Required, but never shown

































            Required, but never shown














            Required, but never shown












            Required, but never shown







            Required, but never shown







            Popular posts from this blog

            Quarter-circle Tiles

            build a pushdown automaton that recognizes the reverse language of a given pushdown automaton?

            Mont Emei